Proof: if A is subset of B then closure of A is subset of closure of B

  • Thread starter Thread starter chessbrah
  • Start date Start date
  • Tags Tags
    closure Proof
Click For Summary
The discussion revolves around proving that if A is a subset of B, then the closure of A is a subset of the closure of B. Participants suggest using proof by contradiction and emphasize the importance of defining "closure" clearly, with a common definition being the smallest closed set containing A. It is noted that if A is contained in a closed set, then the closure of A will also be contained in that closed set. The thread was moved to the appropriate category as it was deemed related to homework rather than number theory. Clarifying definitions is highlighted as a crucial step in simplifying the proof process.
chessbrah
Messages
2
Reaction score
0
I'm thinking proof by contradiction but I can't seem to get anywhere.
 
Physics news on Phys.org
Think about limit points. x is in the closure of A, so it is a limit point of A...
 
You should probably state the definition of "closure" that you would like to use, and maybe also your definition of "closed". I like the definition that says that the closure of E is the smallest closed set that contains E. If we use that definition, the proof is very simple.

I see that you're new here. If you're wondering why we don't just tell you the complete solution, it's because the forum rules tell us not to do that when someone posts a textbook-style question. We are only allowed to give you hints, and tell you if you're doing something wrong.
 
It might be easier to prove that if A is contained in a closed set (here the closure of B, which follows since B is contained in the closure of B and A is contained in B), then the closure of A is contained in the same closed set.
 
May I ask why this thread is here? It isn't number theory, and it looks like homework to me.
 
PAllen said:
May I ask why this thread is here? It isn't number theory, and it looks like homework to me.
I agree. I have moved the thread to "Homework and Coursework- Calculus and Beyond".

I also agree with Fredrik that you need to state what definition of closure you are using. One common definition is that the closure of a set, A, is the smallest closed set that contains A. Using that definition this theorem is pretty close to trivial.
 
Question: A clock's minute hand has length 4 and its hour hand has length 3. What is the distance between the tips at the moment when it is increasing most rapidly?(Putnam Exam Question) Answer: Making assumption that both the hands moves at constant angular velocities, the answer is ## \sqrt{7} .## But don't you think this assumption is somewhat doubtful and wrong?

Similar threads

  • · Replies 1 ·
Replies
1
Views
2K
  • · Replies 16 ·
Replies
16
Views
5K
  • · Replies 7 ·
Replies
7
Views
2K
  • · Replies 6 ·
Replies
6
Views
3K
Replies
1
Views
2K
  • · Replies 2 ·
Replies
2
Views
1K
  • · Replies 7 ·
Replies
7
Views
1K
  • · Replies 10 ·
Replies
10
Views
2K
  • · Replies 8 ·
Replies
8
Views
2K
  • · Replies 4 ·
Replies
4
Views
1K